Simpson's Rule/Trapezoidal Approximation - Error rate help

Click For Summary
The discussion centers on calculating the error rates for the Trapezoidal and Simpson's Rule approximations of the integral of sin(x) from 0 to π. While the calculations for the approximations yield correct results, the participant mistakenly concludes that the maximum values of the second and fourth derivatives are zero, leading to an erroneous error of zero. However, the correct approach requires finding the maximum absolute values of these derivatives over the interval [0, π], which are not zero. The book provides specific error values for both methods, indicating that the participant's misunderstanding lies in the evaluation of M, the maximum derivative values. This highlights the importance of correctly identifying maximum values in error analysis for numerical integration methods.
Asphyxiated
Messages
263
Reaction score
0

Homework Statement



\int^{ \pi}_{0} sin(x)dx \;\;\;\;\;\;\;\; dx=\frac{ \pi}{2}

Homework Equations



Trapezoidal Approximation:

|f''(x)| \leq M \;\;\;\;\; for \;\;\;\;\; a \leq x \leq b

\frac {b-a}{12}(M)(dx)^{2} = Error

Simpson's Rule:

|f^{(4)}(x)| \leq M \;\;\;\;\; for \;\;\;\;\; a \leq x \leq b

\frac{b-a}{180}(M)(dx)^{4} = Error

The Attempt at a Solution



Ok so I have found the correct estimations using both methods easily, the trapezoidal approximation is: 1.5708 and Simpson's Rule is: 2.0944, those numbers check out in the back of the book, but when it comes to finding the error I think that it should be 0 because the max (M) is zero for both the second and fourth derivative but the book says otherwise. Heres what I did:

y=sin(x)

y'=cos(x)

y''=-sin(x)

y^{(3)}= -cos(x)

y^{(4)}= sin(x)

and Trapezoidal Rule using y'' is:

|y''( \pi)|=0

and

|y''(0)|=0

and that follows the same for y^{(4)} so M is 0 and thus the entire equation is 0 and Error = 0 but the book states that the error for the trapezoidal approximation is:

\frac { \pi^{3}}{48} \;\;\;\;\; or \;\;\;\; .65

and the Error for Simpson's Rule is:

\frac { \pi^{5}}{2880} \;\;\;\; or \;\;\;\; .1

I don't see how they got this... but I don't think the actual error rate is zero either because if it were then the trapezoidal and simpson approximation would be exactly equal, so where did I go wrong?
 
Physics news on Phys.org
Asphyxiated said:

Homework Statement



\int^{ \pi}_{0} sin(x)dx \;\;\;\;\;\;\;\; dx=\frac{ \pi}{2}

Homework Equations



Trapezoidal Approximation:

|f''(x)| \leq M \;\;\;\;\; for \;\;\;\;\; a \leq x \leq b

\frac {b-a}{12}(M)(dx)^{2} = Error

Simpson's Rule:

|f^{(4)}(x)| \leq M \;\;\;\;\; for \;\;\;\;\; a \leq x \leq b

\frac{b-a}{180}(M)(dx)^{4} = Error

The Attempt at a Solution



Ok so I have found the correct estimations using both methods easily, the trapezoidal approximation is: 1.5708 and Simpson's Rule is: 2.0944, those numbers check out in the back of the book, but when it comes to finding the error I think that it should be 0 because the max (M) is zero for both the second and fourth derivative but the book says otherwise.
f''(x) = -sin(x) and f(4)(x) = sin(x). The maximum value of the absolute values of these functions is not zero. What you're looking for is the maximum value over the entire interval [0, pi].
Asphyxiated said:
Heres what I did:

y=sin(x)

y'=cos(x)

y''=-sin(x)

y^{(3)}= -cos(x)

y^{(4)}= sin(x)

and Trapezoidal Rule using y'' is:

|y''( \pi)|=0

and

|y''(0)|=0

and that follows the same for y^{(4)} so M is 0 and thus the entire equation is 0 and Error = 0 but the book states that the error for the trapezoidal approximation is:

\frac { \pi^{3}}{48} \;\;\;\;\; or \;\;\;\; .65

and the Error for Simpson's Rule is:

\frac { \pi^{5}}{2880} \;\;\;\; or \;\;\;\; .1

I don't see how they got this... but I don't think the actual error rate is zero either because if it were then the trapezoidal and simpson approximation would be exactly equal, so where did I go wrong?
 
Question: A clock's minute hand has length 4 and its hour hand has length 3. What is the distance between the tips at the moment when it is increasing most rapidly?(Putnam Exam Question) Answer: Making assumption that both the hands moves at constant angular velocities, the answer is ## \sqrt{7} .## But don't you think this assumption is somewhat doubtful and wrong?

Similar threads

Replies
3
Views
2K
  • · Replies 11 ·
Replies
11
Views
3K
  • · Replies 3 ·
Replies
3
Views
2K
  • · Replies 40 ·
2
Replies
40
Views
4K
  • · Replies 2 ·
Replies
2
Views
1K
  • · Replies 2 ·
Replies
2
Views
2K
  • · Replies 5 ·
Replies
5
Views
2K
  • · Replies 4 ·
Replies
4
Views
2K
Replies
3
Views
2K
  • · Replies 7 ·
Replies
7
Views
2K